1
JEE Main 2024 (Online) 6th April Evening Shift
MCQ (Single Correct Answer)
+4
-1

If $$z_1, z_2$$ are two distinct complex number such that $$\left|\frac{z_1-2 z_2}{\frac{1}{2}-z_1 \bar{z}_2}\right|=2$$, then

A
either $$z_1$$ lies on a circle of radius $$\frac{1}{2}$$ or $$z_2$$ lies on a circle of radius 1.
B
$$z_1$$ lies on a circle of radius $$\frac{1}{2}$$ and $$z_2$$ lies on a circle of radius 1.
C
either $$z_1$$ lies on a circle of radius 1 or $$z_2$$ lies on a circle of radius $$\frac{1}{2}$$.
D
both $$z_1$$ and $$z_2$$ lie on the same circle.
2
JEE Main 2024 (Online) 5th April Evening Shift
MCQ (Single Correct Answer)
+4
-1
Change Language

Let $$S_1=\{z \in \mathbf{C}:|z| \leq 5\}, S_2=\left\{z \in \mathbf{C}: \operatorname{Im}\left(\frac{z+1-\sqrt{3} i}{1-\sqrt{3} i}\right) \geq 0\right\}$$ and $$S_3=\{z \in \mathbf{C}: \operatorname{Re}(z) \geq 0\}$$. Then the area of the region $$S_1 \cap S_2 \cap S_3$$ is :

A
$$\frac{125 \pi}{24}$$
B
$$\frac{125 \pi}{6}$$
C
$$\frac{125 \pi}{12}$$
D
$$\frac{125 \pi}{4}$$
3
JEE Main 2024 (Online) 5th April Morning Shift
MCQ (Single Correct Answer)
+4
-1
Change Language

Consider the following two statements :

Statement I: For any two non-zero complex numbers $$z_1, z_2,(|z_1|+|z_2|)\left|\frac{z_1}{\left|z_1\right|}+\frac{z_2}{\left|z_2\right|}\right| \leq 2\left(\left|z_1\right|+\left|z_2\right|\right) \text {, and }$$

Statement II : If $$x, y, z$$ are three distinct complex numbers and $$\mathrm{a}, \mathrm{b}, \mathrm{c}$$ are three positive real numbers such that $$\frac{\mathrm{a}}{|y-z|}=\frac{\mathrm{b}}{|z-x|}=\frac{\mathrm{c}}{|x-y|}$$, then $$\frac{\mathrm{a}^2}{y-z}+\frac{\mathrm{b}^2}{z-x}+\frac{\mathrm{c}^2}{x-y}=1$$.

Between the above two statements,

A
both Statement I and Statement II are incorrect.
B
Statement I is correct but Statement II is incorrect.
C
Statement I is incorrect but Statement II is correct.
D
both Statement I and Statement II are correct.
4
JEE Main 2024 (Online) 4th April Evening Shift
MCQ (Single Correct Answer)
+4
-1

The area (in sq. units) of the region $$S=\{z \in \mathbb{C}:|z-1| \leq 2 ;(z+\bar{z})+i(z-\bar{z}) \leq 2, \operatorname{lm}(z) \geq 0\}$$ is

A
$$\frac{7 \pi}{4}$$
B
$$\frac{3 \pi}{2}$$
C
$$\frac{7 \pi}{3}$$
D
$$\frac{17 \pi}{8}$$
JEE Main Subjects
EXAM MAP
Medical
NEET
Graduate Aptitude Test in Engineering
GATE CSEGATE ECEGATE EEGATE MEGATE CEGATE PIGATE IN
CBSE
Class 12